[MD-sorular] Yılbaşı Sorusu

E. Mehmet Kıral luzumi at gmail.com
12 Şub 2009 Per 18:14:12 EET


Tekrar Merhaba,

Geçen postayla bu posta arasında okuduğum bir makalede bu sorunun çok çok
özel bir halinin çözümünü buldum. Bu da sorunuzun ne kadar zor olduğunu
gösteriyor.

Goldston, Graham, Pintz ve Cem Yalçın Yıldırım'ın yazdığı "Small gaps
between almost primes, the parity problem and some conjectures of Erdös on
consecutive integers" makalesinde cevap var. Bu makaleyi Cem Yalçın
Yıldırım'ın internet sitesinde bulabilirsiniz.

Makalede sizin sorunuzun k = 2 ve m >= 4  durumu çözülüyor. Yani \omega(n) =
\omega(n+1) = A olan sonsuz tane n olduğu kanıtlanıyor.

Daha önce sonsuz tane omega(n) = omega(n+1) olduğu kanıtlanmış. (Böyle
sonsuz sayı olup olmadığı Erdös'ün bir sorusu.) Ancak bu makale omega'nın
aldığı değeri belirterek bu sonucu elde ediyor.
Daha başka sonuçlar da elde ediliyor.

Makale elementer, tüm sonuçlar daha öncek bir makalede kanıtladıkları hiç de
elementer olmayan bir ana teoremden çıkıyor.

Aslında sizin sorunuz bu eşitliğin sağlandığı n'lerin varlığı idi. Ancak
sonlu tane n'nin varlığı analitik sayı teorisi bakış açısından çok bir şey
ifade etmez (başka açılarından edebilir), zira bu eşitlik kazara meydana
gelmiş olabilir.

Gördüğünüz gibi sorunun k = 2 hali bile ne kadar yeni çözülüyor (makale 2008
Mart tarihli). Sanırım bu sorduğunuz sorunun zorluğuna delalet.



2009/1/1 E. Mehmet Kıral <luzumi at gmail.com>

> Bu çok çok zor bir problem, olsa gerek.
>
> Prensip olarak hem çarpanlarla ve hem de ardışıklıkla (toplamayla) ilgili
> problemler çok zor oluyorlar. Zaten problemin bir benzerinin ikiz asal
> problemi olduğuna dikkatinizi çekerim. k = 2 ve m = 1 alıyorsunuz. Tabii
> ardışık iki sayı birden asal olamayacağından mecburen ardışık iki sayı asal
> olamayacağından ortadakini atıyoruz.
>
> Siz istediğiniz özelliği sağlayan bir sayının "bulunmasını" istemişsiniz.
> Tahminimce bu çok zor bir sorudur. Onun yerine böyle bir sayının varlığını
> kanıtlamak daha ulaşılabilirdir. Bu varlığı kanıtlamanın yolu da, bu tip
> sayıların yoğunluğunun sıfırdan büyük olduğunu kanıtlamaktan geçiyor (vur
> dediniz, öldürdük).
>
> Soru aslında (büyük) omega fonksiyonunun sabitlendiği bir aralık arıyor.
> http://mathworld.wolfram.com/PrimeFactor.html
> Oradan da görülebileceği üzere
> Büyük omega fonksiyonu loglog n mertebesinde büyüyor ve büyürken de
> varyansı yine loglog n.
>
> Bize çözümün imkansızlığını değil de çözüm noktalarının nerelerde olmasının
> daha olası olduğunu söylüyor bu oran. Aynı zamanda istediğiniz problemi
> çözen noktaların sayısına asimptotik bir üstsınır da getiriyor.
> Tabii bizim asıl istediğimiz üstsınır değil, altsınır. Altsınırın sonsuza
> gittiğini gösterelim ki en azından bir tane çözüm olduğunu gösterebilelim.
>
> İkiz asalların yoğunluğuyla ilgili olarak da elimizde olanlar üstsınırlar,
> altsınır değil.
>
> İstediğiniz türde sayıların olduğunu kanıtlamak muhtemelen çok çok çok çok
> zor bir soru (diğer zor sorulara olan benzerliğinden çıkarıyorum bunu) ancak
> yine muhtemelen böyle sayılar varlar. Çünkü;
> Eğer, asal sayılarla ilgili bir gerçek basit denkliklerden dolayı imkansız
> değilse, gerçekleşiyor. Hem de sonsuz kere.
> Bu matematiksel bir ifade değil tabii ki, şimdiye kadar gözlemlenenden
> yapılan bir çıkarım.
>
> 2009/1/1 dede <dede_47 at mynet.com>
>
>>  Merhaba;
>>
>> Geçen akşam "eski matematik defterlerimi" karıştırırken;
>>
>> "Zor sorular" dosyasında  "çözemedim" notlu bir soruya tekrar takıldım.
>>
>> Nerden bu soruyu almışım, bilemiyorum. İhmal işte yazmamışım.
>>
>> Soru şöyle: "Öyle bir n tamsayısı bulunuz ki:
>>
>>  n, n+1, n+2, n+3, n+4, n+5, n+6 tamsayılarının her birinde,
>>
>> Tam 3 adet asal çarpan olsun!"
>>
>> (Asal çarpanlar, birbirini aynı veya farklı olabilir)!
>>
>> Tekrar uğraştım, yine çözemedim!
>>
>> Canım sıkıldı,"köleme"  soruyu programladım!
>>
>> "Kölem", 5 dakika "çalıştı"; ekranda bir sayı: "n=211673";
>>
>> Bu sayı sorunun verilerini karşılıyordu;
>>
>> Ama "kölem" çözmüştü, ben değil!
>>
>> Belki daha başka sayılarda vardı, koşulları sağlayacak!
>>
>> Bilmiyorum:Zira "kölemi" durdurdum! Daha sonra şeytan dürttü:
>>
>> Bu soruyu genellesek dedim; yani:
>>
>> "Öyle bir n tamsayı bununuz ki; kendisi de dahil, sıralı olarak gelecek;
>>
>> k adet tamsayının her birinde  tam m adat asal çarpan bulunsun!
>>
>> (Asal çarpanlar birbirinden farkı veya eşit olabilir.
>>
>> Yukarda ki özel halde n=211673, k=7, m=3)
>>
>> Genellediğim bu soruyu değil çözmek,yanına dahi yaklaşamadım!
>>
>> Madem çözemedim, paylaşayım dedim:
>>
>> "Sayılar kuramını" iyi bilen birisi...belki... genelini değil ama özelini...
>>
>> "Kölesiyle" değil, çözümün açıkça  görüldüğü; "biyolojik" olarak,
>>
>> Çözebilecek bir "dost" çıkar! "Onun" şimdiden, "aklına/eline sağlık" der...
>>
>> Cümle "dostların" yeni yılını kutlar....ve,
>>
>> Herkese mutluluklar dilerim.
>>
>>
>>
>> A.Kadir Değirmencioğlu
>>
>> _______________________________________________
>> MD-sorular e-posta listesi
>> sorular at matematikdunyasi.org
>> http://lists.math.bilgi.edu.tr/cgi-bin/mailman/listinfo/md-sorular
>>
>
>
>
> --
> Eren Mehmet Kıral
>



-- 
Eren Mehmet Kıral
-------------- sonraki bölüm --------------
Bir HTML eklentisi temizlendi...
URL: http://lists.math.bilgi.edu.tr/pipermail/md-sorular/attachments/20090212/ddb4981f/attachment.htm 


MD-sorular mesaj listesiyle ilgili daha fazla bilgi